You are on page 1of 9

PHY3063 Spring 2006 R. D.

Field

PHY 3063 Final Exam Solutions


Problem 1 (35 points): Consider an particle with mass m
confined within an infinite square well defined by Infinite Square Well

V(x) = 0 for 0 < x < L, V = +infinity V = +infinity


V(x) = +∞ otherwise.
(a) (5 points): Using Schrödinger’s equation calculate the
allowed stationary state eigenfunctions ψn(x), where the complete
wavefunctions are given by Ψn ( x, t ) = ψ n ( x )e n . Normalize
− iE t / h
0 L x

the eigenfunctions so that the probability of finding the particle somewhere in the box is one.
2
Answer: ψ n ( x) = sin( nπx / L)
L
Solution: For the region outside of 0 < x < L ψ ( x) = 0 and inside the region
h 2 d 2ψ ( x) d 2ψ ( x) h 2k 2
− = Eψ ( x ) or = − k 2
ψ ( x ) with E =
2me d 2 x d 2x 2me
The most general solution is of the form
ψ ( x) = A sin(kx) + B cos(kx) .
The boundary condition at x = 0 givesψ (0) = B = 0 and the boundary condition at x = L gives
ψ ( L) = A sin(kL) = 0 which implies that kL = nπ with n = 1, 2, 3,… Thus,
n 2π 2h 2
ψ n ( x) = A sin(nπx / L) with En = . The normalization is arrived at by requiring that
2mL2
+∞ L nπ nπ
LA2 LA2  y sin(2 y )  LA2
∫ψ ( x)ψ n ( x)dx = 1 = A ∫0 sin (nπx / L)dx = nπ ∫0 sin ( y)dy = nπ  2 − 4  0 = 2
∗ 2 2 2
n
−∞

Thus, A = 2 / L . These states are called stationary because the probability density and all the
expectation values are independent of time.
(b) (5 points): Show that the allowed energy levels of the system are, En = E0 n2,
where E0 = h 2π 2 /(2mL2 ) is the ground state energy and n = 1, 2, 3, …. Why is n = 0 excluded as
a possible energy level?
n 2π 2 h 2 π 2h 2
Solution: We see from above that En = = n E0 with E1 = E0 =
2
. The state with
2mL2 2mL2
n = 0 correstions to ψ0(x) = 0 which is not normalizable.
(c) (10 points): Consider the operator, O = (x)op(px)op (i.e. the product of the position operator
times the momentum operator). Is the operator O hermitian? Calculate the expectation value of
the operator Q for the nth stationary state (i.e. calculate <ψn|O|ψn>).
Answer: O is not hermitian and < xpx > n = ih / 2 .
Solution: We see that O ↑ = ( xpx )↑ = ( p x )↑ x ↑ = px x ≠ xpx since (x)op and (px)op do not commute.
Also,

Final Exam Solutions Page 1 of 9 April 25, 2006


PHY3063 Spring 2006 R. D. Field

+∞ +∞
dψ n ( x )
< xpx > n = ∫ψ n∗ ( x)( xpx )opψ n ( x)dx = −ih ∫ ψ n∗ ( x) x dx
−∞ −∞
dx
 2  nπ   2  nπ 
L L
− ih   ∫ x sin(nπx / L) cos(nπx / L)dx = −ih   ∫ x sin( 2nπx / L)dx =
 L  L  0  L  2 L  0
2 2 nπ
 2  nπ  L   1   − 2 nπ  i h
(sin y − y cos y ) 0 = −ih
2 nπ
= −ih    ∫ y sin( y )dy = −ih =
 L  2 L  2nπ  0  4 nπ   4 nπ  2
where I used sin θ cosθ = 12 sin 2θ and I let y = 2nπ/L.
(d) (15 points): Suppose the particle in this infinite square well has an initial wave function at
t = 0 given by
Ψ ( x,0) = A sin 2 (πx / L) .
What is the normalization A? If you measure the energy of this particle, what is the probability
that you will measure the ground state energy E0?
8 768
Answer: A = and P1 = ≈ 0.96067
3L 81π 2
Solution: The normalization is arrived at by requiring that
+∞ L π
LA2
∫ Ψ ( x,0)Ψ( x,0)dx = 1 = A ∫ sin (πx / L)dx = ∫ sin ( y)dy
∗ 2 4 4

−∞ 0
π 0
π
LA2  3 y sin(2 y ) sin(4 y )  LA2 3π 3LA2
=  − +  = =
π  8 4 32  0 π 8 8
8
and hence A = . The overlap of Ψ(x,0) with ψ1(x) is given by
3L
+∞ L π
2 8 2 8 L
c1 =< ψ 1 | Ψ >= ∫ψ 1 ( x)Ψ ( x,0)dx =

∫ sin 3 (πx / L)dx = ∫ sin 3 ( y )dy
−∞
L 3L 0 L 3L π 0
π
2 8 L  cos3 ( y )  2 8 L 1 1  4 16 16 3
=  − cos( y )  =  − + 1 − + 1 = =
L 3L π  3 0 L 3L π  3 3  3π 3 9π
and the probability, P1, of measuring E0 = E1 is
2
 16 3  768
P1 =| c1 | = 
2
 =
 ≈ 0.96067 .
 9π  81π
2

Final Exam Solutions Page 2 of 9 April 25, 2006


PHY3063 Spring 2006 R. D. Field

Problem 2 (35 points): Consider the case of two non-interacting particles Two Particles in a Box
both with mass m in a one-dimensional infinite square well given by
V(x) = 0 for 0 < x < L, and V(x) = ∞. For one particle we know that the
stationary states of Schrödinger’s equation are given by ψ(x2)
Ψn ( x, t ) = ψ n ( x)e −iE n t / h and En = n 2 E0 , and n is a positive integer and
π 2h 2 ψ(x1)
E0 = and where you calculated ψn(x) in the previous problem. For
2mL2
two (non-interacting) particles we look for a solution of the form
Ψ ( x1 , x 2 , t ) = ψ ( x1 , x 2 )e − iEt / h = ψ ( x1 )ψ ( x 2 )e − iEt / h with 0 L
2 2
(p ) (p )
+ x 1
=E. x 2

2m 2m
(a) (15 points): Show that ψ αβ ( x1 , x2 ) = ψ α ( x1 )ψ β ( x2 ) , is a solution to the two particle non-
interaction Schrödinger equations, where α and β are positive integers and ψα(x) and ψβ(x) are
the one particle stationary state solutions from the previous problem. Show that the allowed
energy levels are given by Eαβ = (α 2 + β 2 ) E0 . This solution corresponds to the case where the
two particles are distinguishable. The probability of finding both particles on the left side of the
box (0 < x < L/2) is given by
L/2 L/2
Pαβ = ∫ ∫ | ψ αβ ( x , x ) |
LL 2
1 2 dx1dx2 .
0 0

Calculate PαβLL for the state where α = 1 and β = 2 (i.e. 1st excited state) for the distinguishable
case.
1
Answer: ( P12LL ) D =
4
Solution: In this case we have
2
ψ αβ ( x1 , x2 ) = ψ α ( x1 )ψ β ( x2 ) = sin(απ x1 / L ) sin( βπ x2 / L )
L
4
ραβ
D
( x1 , x 2 ) =| ψ αβ ( x1 , x 2 ) | 2 = 2
sin 2 (απx1 / L) sin 2 ( βπ x 2 / L )
L
and
L/2L/2
4
(P ) = 2 ∫ ∫ sin (πx1 / L) sin 2 (2πx2 / L)dx1dx2
LL 2
12 D
L 0 0
L/2 L/2
2 2  1  1  1
∫ sin (πx1 / L)dx1 ∫ sin (2πx2 / L)dx2 =    =
2 2

L 0
L 0  2  2  4
where I used
nπ / 2 nπ / 2
 2  nπ  1
L/2
2 2 L  2  L  y sin(2 y ) 
∫0 sin (nπx / L)dx = L  nπ  ∫0 sin ( y)dx = L  nπ  2 − 4  0 =  nπ  4  = 2
2 2

L
and I let y = nπ/L.
(b) (10 points): For two identical bosons (i.e. particles with integral spins) we must use the
symmetric wavefunction

Final Exam Solutions Page 3 of 9 April 25, 2006


PHY3063 Spring 2006 R. D. Field

ψ αβ
S
( x1 , x 2 ) =
1
(ψ αβ ( x1 , x 2 ) + ψ αβ ( x 2 , x1 ) ) (α ≠ β symmetric under 1↔2)
2
1
ψ αα
S
( x1 , x 2 ) = (ψ αα ( x1 , x2 ) + ψ αα ( x2 , x1 ) ) (α = β symmetric under 1↔2)
2
Calculate PαβLL for the state where α = 1 and β = 2 (i.e. 1st excited state) for two indistinguishable
bosons.
2
1  4  1
Answer: ( P ) = +   ≈ + 0.18 ≈ 0.43
LL

4  3π 
12 BE
4
Solution: In this case we have
2
ψ αβ
S
( x1 , x2 ) = [sin(απx1 / L) sin( βπ x2 / L ) + sin( βπ x1 / L) sin(απx2 / L )] (α ≠ β)
L
2
ψ αα
S
( x1 , x2 ) = sin(απx1 / L ) sin(απx2 / L )
L
and

ραβ
BE
( x1 , x2 ) = ραβ
classical
( x1 , x2 ) + ραβ
int
( x1 , x2 ) (α ≠ β)
where
ραβ
int
( x1 , x2 ) ≡ Re (ψ αβ ( x1 , x2 )ψ βα

( x1 , x2 ) ) (α ≠ β)
and
ραα
BE
( x1 , x2 ) = ραα
classical
( x1 , x2 ) .
We see that
ραβ
classical
( x1 , x2 ) =
L
2
2
(
sin 2 (απx1 / L) sin 2 ( βπx2 / L) + sin 2 ( βπx1 / L) sin 2 (απx2 / L) )
4
ραβ
int
( x1 , x2 ) = 2 (sin(απx1 / L) sin(βπx1 / L) sin(απx2 / L) sin( βπx2 / L) )
L
and
ραβ
classical
( x1 , x2 ) = 12 ( ραβ
D
( x1 , x2 ) + ραβ
D
( x2 , x1 )) .
We see that
L/2 L/2 L/2 L/2 L/2 L/2
( P12LL ) BE = ∫ ∫ ρ12 ( x1, x2 )dx1dx2 = ∫ ∫ ρ12 ( x1, x2 )dx1dx2 + ∫ ∫ρ
BE classical int
12 ( x1 , x2 )dx1dx2
0 0 0 0 0 0
2
1  4  1
= +   ≈ + 0.18 ≈ 0.43
4  3π  4
where I used

Final Exam Solutions Page 4 of 9 April 25, 2006


PHY3063 Spring 2006 R. D. Field

L/2 L/2 L/2 L/2


4
∫ ∫0 ρ ( x1, x2 )dx1dx2 = L2 ∫ ∫ sin(πx / L) sin(2πx / L) sin(πx / L) sin(2πx2 / L)dx1dx2
int
12 1 1 2
0 0 0
L/2 L/2 2
2 2  4  4   4 
=
L ∫0 sin(πx1 / L) sin(2πx1 / L)dx1 L ∫0 sin(πx2 / L) sin(2πx2 / L)dx2 =  − 3π  − 3π  =  3π 
and
L/2 L/2
2 1
L ∫ sin(πx / L) sin(2πx / L)dx =
0
L ∫ (cos(3πx / L) − cos(πx / L))dx =
0
L/2 L/2
1 1 L L
∫ cos(3πx / L)dx − L ∫ cos(πx / L)dx = 3πL (sin( y)) (sin( y) ) π0 / 2
3π / 2

L 0 0
0
πL
1 1 4
=− − =−
3π π 3π

(c) (10 points): For two identical fermions (i.e. particles with half-integral spins) we must use
the symmetric wavefunction

ψ αβA ( x1 , x 2 ) = (ψ αβ ( x1 , x2 ) −ψ αβ ( x2 , x1 ) ) (antisymmetric under 1↔2)


1
2
Calculate PαβLL for the state where α = 1 and β = 2 (i.e. ground state) for two indistinguishable
fermions.
2
1  4  1
Answer: ( P12 ) FD = −   ≈ − 0.18 ≈ 0.07
LL

4  3π  4
Solution: In this case we have
2
ψ αβA ( x1 , x2 ) = [sin(απx1 / L ) sin( βπ x2 / L) − sin( βπ x1 / L) sin(απx2 / L )]
L
and

ραβ
FD
( x1 , x2 ) = ραβ
classical
( x1 , x2 ) − ραβ
int
( x1 , x2 )
where
ραβ
int
( x1 , x2 ) ≡ Re (ψ αβ ( x1 , x2 )ψ βα

( x1 , x2 ) ) (α ≠ β)
and we get
L/2L/2 L/2L/2 L/2L/2
= ∫ ∫ρ ( x1 , x2 )dx1dx2 = ∫ ∫ρ ( x1 , x2 )dx1dx2 − ∫ ∫ρ
LL FD classical int
(P )
12 FD 12 12 12 ( x1 , x2 )dx1dx2
0 0 0 0 0 0
2
1  4  1
= −   ≈ − 0.18 ≈ 0.07
4  3π  4

Final Exam Solutions Page 5 of 9 April 25, 2006


PHY3063 Spring 2006 R. D. Field

Problem 3 (30 points): Suppose we have two vector operators


r r r r
( J1 )op and ( J 2 )op with [( J1 )op , ( J 2 )op ] = 0
and each of the vectors obey the same SU(2) “lie algebra”
[( J1i )op , ( J1 j )op ] = iε ijk ( J1k )op and [( J 2i )op , ( J 2 j )op ] = iε ijk ( J 2 k )op .
The states |j1m1> are the eigenkets of ( J12 ) op and ( J1z )op and the states |j2m2> are the eigenkets
of ( J 22 ) op and ( J 2 z )op as follows:

( J12 )op | j1m1 >= j1 ( j1 + 1) | j1m1 > ( J 22 )op | j2 m2 >= j2 ( j2 + 1) | j2 m2 >


( J1z )op | j1m1 >= m1 | j1m1 > ( J 2 z )op | j2 m2 >= m2 | j2 m2 >
Also we know that
( J1± )op | j1m1 >= j1 ( j1 + 1) − m1 (m1 ± 1) | j1m1 ± 1 >
( J 2± )op | j2 m2 >= j2 ( j2 + 1) − m2 (m2 ± 1) | j2 m2 ± 1 >
± ±
where ( J1 )op = ( J1x )op ± i ( J1 y )op and ( J 2 )op = ( J 2 x )op ± i ( J 2 y )op . Now consider the vector
sum of the two operators,
r r r
( J )op = ( J1 )op + ( J 2 )op or ( J i )op = ( J i1 )op + ( J i 2 )op for i = 1,2, 3.
(a) (5 points): Show that
r r r r r r r r
( J 2 )op = ( J )op ⋅ ( J )op = ( J1 + J 2 )op ⋅ ( J1 + J 2 )op = ( J12 )op + ( J 22 )op + 2( J1 )op ⋅ ( J 2 )op
= ( J12 )op + ( J 22 )op + ( J1+ )op ( J 2− )op + ( J1− )op ( J 2+ )op + 2( J1z )op ( J 2 z )op
Solution: First we note that
J1x = 12 ( J1+ + J1− ) J 2 x = 12 ( J 2+ + J 2− )
and
J1 y = 1
2i
( J1+ − J1− ) J2y = 1
2i
( J 2+ − J 2− )
Hence,
r r
J 2 = J12 + J 22 + 2 J1 ⋅ J 2 = J12 + J 22 + 2 J1x J 2 x + 2 J1 y J 2 y + 2 J1z J 2 z
= J12 + J 22 + 12 ( J1+ + J1− )( J 2+ + J 2− ) − 12 ( J1+ − J1− )( J 2+ − J 2− ) + 2 J1z J 2 z
= J12 + J 22 + J1+ J 2− + J1− J 2+ + 2 J1z J 2 z
(b) (5 points): Evaluate the following in SU(2).
3×2= 4×3= 5×3=
5×4= 2×3×4=
Answers:
3 × 2 = 4 + 2 (j1 = 1, j2 = ½, j = 3/2, 1/2)
4 × 3 = 6 + 4 + 2 (j1 = 3/2, j2 = 1, j = 5/2, 3/2, 1/2)
5 × 3 = 7 + 5 + 3 (j1 = 2, j2 = 1, j = 3, 2, 1)
5 × 4 = 8 + 6 + 4 + 2 (j1 = 2, j2 = 3/2, j = 7/2, 5/2, 3/2, 1/2)
2 × 3 × 4 = 2 × (6 + 4 + 2) = 2×6 + 2×4 + 2×2 = 7 + 5 +5 + 3 + 3 + 1

Final Exam Solutions Page 6 of 9 April 25, 2006


PHY3063 Spring 2006 R. D. Field

where I used
2 × 6 = 7 + 5 (j1 = 1/2, j2 = 5/2, j = 3, 2)
2 × 4 = 5 + 3 (j1 = 1/2, j2 = 3/2, j = 2, 1)
2 × 2 = 3 + 1 (j1 = 1/2, j2 = 1/2, j = 1, 0)
(c) (20 points): Now consider the case where j1 = 1 and j2 = ½ (i.e. 3 × 2) and define the states as
follows:
| Y11 >1 =| 11 >
|↑>2 =| 12 12 >
| Y10 >1 =| 10 > and
|↓>2 =| 12 − 12 >
| Y1−1 >1 =| 1 − 1 >
Now consider the two superposition states
1 2 2 1
| + >≡ | Y11 >1|↓> 2 + | Y10 >1|↑> 2 and | − >≡ | Y11 >1|↓> 2 − | Y10 >1|↑> 2 .
3 3 3 3
Calculate the following and express your answer in terms of |±>:
(1) J z | ± > (2) J1 | ± > (3) J 2 | ± >
2 2

+ − − +
(4) 2 J1z J 2 z | ± > (5) ( J1 J 2 + J1 J 2 ) | ± > (6) J | ± >
2

Are the states | ± > eigenstates of the J and Jz and if so what are their eigenvalues?
Answer:
(1) J z | ± >= 12 | ± > (2) J1 | ± >= 2 | ± > (3) J 2 | ± >= 34 | ± >
2 2

(
(4) 2 J1z J 2 z | ± >= − 13 (( 32 m 12 ) | ± > + 2 | m > )
(5) ( J1+ J 2− + J1− J 2+ ) | ± >= ± 43 | ± > + 3
2
|m>
(6) J | ± >= ( 94 ± 64 ) | ± >
2

The state |+> correspomds to j = 3/2 m = ½ and |-> corresponds to j = ½ and m = ½.


Solution: We know that
J12 | Y11 >1 = J12 | 11 >= 1(1 + 1) | 11 >= 2 | Y11 >1 J 22 |↑> 2 = J 22 | 12 12 >= 12 ( 12 + 1) | 12 12 >= 34 |↑> 2
J12 | Y10 >1 = J12 | 10 >= 1(1 + 1) | 10 >= 2 | Y10 >1 J 22 |↓> 2 = J 22 | 12 − 12 >= 12 ( 12 + 1) | 12 − 12 >= 34 |↓> 2
J1z | Y11 >1 = J1z | 11 >= 1 | 11 >= 1 | Y11 >1 J 2 z |↑>2 = J 2 z | 12 12 >= 12 | 12 12 >= 12 |↑>2
J1z | Y11 >1 = J1z | 10 >= 0 | 10 >= 0 J 2 z |↓>2 = J 2 z | 12 − 12 >= − 12 | 12 − 12 >= − 12 |↓>2
J1+ | Y11 >1 = J1+ | 11 >= 0
J1+ | Y10 >1 = J1+ | 10 >= 1(1 + 1) + 0(0 + 1) | 11 >= 2 | Y11 >1

J1− | Y11 >1 = J1− | 11 >= 1(1 + 1) − 1(1 − 1) | 10 >= 2 | Y10 >1
J 2+ |↑>2 = J 2+ | 12 12 >= 0
J 2+ |↓>2 = J 2+ | 12 − 12 >= 1
2 ( 12 + 1) + 12 (− 12 + 1) | 12 12 >=|↑> 2
J 2− |↑>2 = J 2− | 12 12 >= 1
2 ( 12 + 1) − 12 ( 12 − 1) | 12 − 12 >=|↓>2
J 2− |↓>2 = J 2− | 12 − 12 >= 0
and hence

Final Exam Solutions Page 7 of 9 April 25, 2006


PHY3063 Spring 2006 R. D. Field

 1 2  1 2
J z | + >= ( J1z + J 2 z ) | Y11 >1|↓> 2 + | Y10 >1|↑> 2  = (1 − 12 ) | Y11 >1|↓> 2 +(0 + 12 ) | Y10 >1|↑> 2 = 12 | + >
 3 3  3 3

 2 1  2 1
J z | − >= ( J1z + J 2 z ) | Y11 >1|↓> 2 − | Y10 >1|↑> 2  = (1 − 12 ) | Y11 >1|↓> 2 −(1 − 12 ) | Y10 >1|↑> 2 = 12 | − >
 3 3  3 3

 1 2  1 2
J12 | + >= J12  | Y11 >1|↓> 2 + | Y10 >1|↑> 2  = (2) | Y11 >1|↓> 2 +(2) | Y10 >1|↑> 2 = 2 | + >
 3 3  3 3
 2 1  2 2
J12 | − >= J12  | Y11 >1|↓> 2 − | Y10 >1|↑> 2  = (2) | Y11 >1|↓> 2 −(2) | Y10 >1|↑> 2 = 2 | − >
 3 3  3 3
 1 2  1 2
J 22 | + >= J 22  | Y11 >1|↓> 2 + | Y10 >1|↑> 2  = ( 34 ) | Y11 >1|↓> 2 +( 34 ) | Y10 >1|↑> 2 = ( 34 ) | + >
 3 3  3 3
 2 1  2 2
J 22 | − >= J 22  | Y11 >1|↓> 2 − | Y10 >1|↑> 2  = ( 34 ) | Y11 >1|↓> 2 −( 34 ) | Y10 >1|↑> 2 = ( 34 ) | − >
 3 3  3 3
We also see that
 1 2 
2 J 1 z J 2 z | + >= 2 J 1 z J 2 z  | Y11 >1 |↓> 2 + | Y10 >1 |↑> 2 
 3 3 
1 2 1
= 2 (1)( − 12 ) | Y11 >1 |↓> 2 + 2( 0)( + 12 ) | Y10 >1 |↑> 2 = − | Y11 >1 |↓> 2
3 3 3

=−
1 1
3 3
( 1
|+ >+ 2|−> =− |+ >+ 2|−>
3
) ( )
and
 2 1 
2 J1z J 2 z | − >= 2 J1z J 2 z  | Y11 >1|↓> 2 − | Y10 >1|↑> 2 
 3 3 
2 1 2
= 2(1)(− 12 ) | Y11 >1|↓> 2 −2(0)(+ 12 ) | Y10 >1|↑> 2 = − | Y11 >1|↓> 2
3 3 3

=−
2 1
3 3
( 1
)
|+ > + 2 |− > = − 2|− > + 2 |+ >
3
( )
and hence,
2 J1z J 2 z | ± >= −
3
(
1 3 1
)
(( 2 m 2 ) | ± > + 2 | m > .
Also,

Final Exam Solutions Page 8 of 9 April 25, 2006


PHY3063 Spring 2006 R. D. Field

 1 2 
( J1+ J 2− + J1− J 2+ ) | + >= ( J1+ J 2− + J1− J 2+ ) | Y11 >1|↓> 2 + | Y10 >1|↑> 2 
 3 3 
1 2 1 2
= 2 | Y10 >1|↑> 2 + 2 | Y11 >1|↓> 2 = 2 | Y11 >1|↓> 2 + | Y10 >1|↑> 2
3 3 3 3
2
3
(
|+>+ 2|−> +
3
2
) ( 2 |+ > −|− > = ) 4
3
|+>+
3
2
|−>
and
 2 1 
( J1+ J 2− + J1− J 2+ ) | − >= ( J1+ J 2− + J1− J 2+ ) | Y11 >1|↓> 2 − | Y10 >1|↑> 2 
 3 3 
2 1 2 1
= 2 | Y10 >1|↑> 2 − 2 | Y11 >1|↓> 2 = − | Y11 >1|↓> 2 +2 | Y10 >1|↑> 2
3 3 3 3

=−
3
2
(
|+>+ 2|−> +
2
3
) ( 4
)
2 |+ > −|− > = − |− > +
3 3
2
|+>
Hence,
4 2
( J 1+ J 2− + J1− J 2+ ) | ± >= ± |±>+ | m >,
3 3
where I used

| Y11 >1|↓> 2 =
1
3
(
|+>+ 2|−> ) and | Y10 >1|↑> 2 =
1
3
( 2 |+ > −|− > . )
We see that
J 2 | ± >= ( J12 + J 22 + J1+ J 2− + J1− J 2+ + 2 J1z J 2 z ) | ± >
3 4
= 2|± > + |± > ± |± > +
4 3 3
2 1
(
| m > − (( 32 m 12 ) | ± > + 2 | m >
3
)
 3 4 1   2 2
=  2 + ± − ( 32 m 12 )  | ± > + − |m >

 4 3 3   3 3 
 3 4 1 1  27 18  9 6
=  2 + ± − ±  | ± >=  ±  | ± >=  ±  | ± >
 4 3 2 6  12 12  4 4
and hence
9 6 15 9 6 3
J 2 | + >==  +  | + >= | + > and J 2 | − >==  −  | − >= | − > .
4 4 4 4 4 4
Thus |+> correspomds to j = 3/2 m = ½ and |-> corresponds to j = ½ and m = ½.

Final Exam Solutions Page 9 of 9 April 25, 2006

You might also like